LSAT and Law School Admissions Forum

Get expert LSAT preparation and law school admissions advice from PowerScore Test Preparation.

 Administrator
PowerScore Staff
  • PowerScore Staff
  • Posts: 8917
  • Joined: Feb 02, 2011
|
#32029
Please post below with any questions!
 ClaudiaK32
  • Posts: 14
  • Joined: May 01, 2017
|
#34950
I chose B for this answer and am unsure for why C is correct.
 Charlie Melman
PowerScore Staff
  • PowerScore Staff
  • Posts: 85
  • Joined: Feb 10, 2017
|
#35147
Hi Claudia,

Answer choice (B) is incorrect because there is one setup where Q pushes the virus to two other computers. If Q is the piece that transmits to R and S, and T is the piece that transmits to Q (as the rules allow), then it's entirely possible to construct a situation that does not violate any of the rules.

But in answer choice (C), you can't satisfy the second-to-last and last rules at the same time, while also making R transmit to two other computers. You can satisfy one of the two, but not both.

I hope this makes sense!
 Ari
  • Posts: 22
  • Joined: Aug 27, 2020
|
#79194
I am having a hard time with this. I also chose B. I tried to work out the problem as Charlie explained where T receives the virus, and transmits it Q, which transmits it to R and S. However, S has to transmit it to one computer and we are left with U and P. P has to receive the virus from T or U, so S must transmit the virus to U, but now where does P go? What am I missing here that would resolve my issue? Otherwise, I do not see how Q would not be correct. I do see why R is correct, but I just can't seem to create a diagram to make Q incorrect. Below is my replication of the diagram described above.

--- R --- P (wrong)
Virus --- T -- Q
--- S --- U
Charlie Melman wrote:Hi Claudia,

Answer choice (B) is incorrect because there is one setup where Q pushes the virus to two other computers. If Q is the piece that transmits to R and S, and T is the piece that transmits to Q (as the rules allow), then it's entirely possible to construct a situation that does not violate any of the rules.

But in answer choice (C), you can't satisfy the second-to-last and last rules at the same time, while also making R transmit to two other computers. You can satisfy one of the two, but not both.

I hope this makes sense!
 Ari
  • Posts: 22
  • Joined: Aug 27, 2020
|
#79196
Never mind! I was reading the 2nd rule to mean that S could only transmit it to one and that was the end, but you could have
S --- U --- P and R does not transmit it to any computer.
Ari wrote:I am having a hard time with this. I also chose B. I tried to work out the problem as Charlie explained where T receives the virus, and transmits it Q, which transmits it to R and S. However, S has to transmit it to one computer and we are left with U and P. P has to receive the virus from T or U, so S must transmit the virus to U, but now where does P go? What am I missing here that would resolve my issue? Otherwise, I do not see how Q would not be correct. I do see why R is correct, but I just can't seem to create a diagram to make Q incorrect. Below is my replication of the diagram described above.

--- R --- P (wrong)
Virus --- T -- Q
--- S --- U
Charlie Melman wrote:Hi Claudia,

Answer choice (B) is incorrect because there is one setup where Q pushes the virus to two other computers. If Q is the piece that transmits to R and S, and T is the piece that transmits to Q (as the rules allow), then it's entirely possible to construct a situation that does not violate any of the rules.

But in answer choice (C), you can't satisfy the second-to-last and last rules at the same time, while also making R transmit to two other computers. You can satisfy one of the two, but not both.

I hope this makes sense!
 kupwarriors9
  • Posts: 73
  • Joined: Jul 01, 2021
|
#88699
Can someone tell me why my thought process was wrong? I had originally thought the answer would be T or U - not R.

U P R Q S T

Thus, couldn't have R transmitted the virus to two other computers (S and T)?
 Robert Carroll
PowerScore Staff
  • PowerScore Staff
  • Posts: 1787
  • Joined: Dec 06, 2013
|
#88981
kup,

It's impossible to R to transmit to S. R and S receive the virus from the same computer. In order for R to transmit to S, R would have to transmit to itself, which is impossible.

Robert Carroll
User avatar
 christinecwt
  • Posts: 74
  • Joined: May 09, 2022
|
#97595
Hi All, can anyone explain why R can send to two more coputers (such as U and P). Many thanks!
User avatar
 bebeg3168
  • Posts: 22
  • Joined: Aug 01, 2022
|
#97960
christinecwt wrote: Mon Oct 03, 2022 6:19 am Hi All, can anyone explain why R can send to two more coputers (such as U and P). Many thanks!
Hi Christinecwt,
I believe the 3rd rule eliminates the possibility. I missed this one too. Hope this helps :-)
User avatar
 Paul Popa
PowerScore Staff
  • PowerScore Staff
  • Posts: 64
  • Joined: Sep 20, 2022
|
#98352
bebeg3168 wrote: Mon Oct 24, 2022 7:26 am
christinecwt wrote: Mon Oct 03, 2022 6:19 am Hi All, can anyone explain why R can send to two more coputers (such as U and P). Many thanks!
Hi Christinecwt,
I believe the 3rd rule eliminates the possibility. I missed this one too. Hope this helps :-)
Hi Christine and Bebe,

A combination of rules 2, 3, and 5 ultimately render it impossible for R to transmit the virus to two computers. We have six computers to play with, and one has to be the computer that transmits the virus to R and S. We also need S to transmit the virus to one (and only one) other computer. These rules force four of the computers into a placement already. We then see from rule 5 that only T or U can send the virus to P, so while we can have R transmit the virus to Q, for example, we would need to build a chain off of S to satisfy that last rule. Hope this helps!

Get the most out of your LSAT Prep Plus subscription.

Analyze and track your performance with our Testing and Analytics Package.